RegistrierenRegistrieren   LoginLogin   FAQFAQ    SuchenSuchen   
Schrödinger Gleichung und Lösungen
 
Neue Frage »
Antworten »
    Foren-Übersicht -> Quantenphysik
Autor Nachricht
volvic
Gast





Beitrag volvic Verfasst am: 19. Jul 2013 12:54    Titel: Schrödinger Gleichung und Lösungen Antworten mit Zitat

Hi

Ich habe eine Frage zu wieso Lösungen der Schrödingergleichung welche die Wahrscheinlichkeit des Ortes des Teilchens im Unendlichen haben.

Wenn ich die Vorlesung richtig verstanden habe, dann sind diese Lösungen nicht möglich, da ein Axiom der QM sagt, dass das Teilchen immer in einem finiten Bereich gefunden werden muss (das Integral 1 ergibt).

Versuchsaufstellungen wie der Doppelspalt oder Poisson Spot Versuch wiedersprechen diesem. An manchen Orten wird das Teilchen nicht zu finden sein auf Grund der Interferenz. Das Integral ist somit nicht 1, da das Teilchen nicht überall zu finden ist?

Vielen dank für eine Antwort
TomS
Moderator


Anmeldungsdatum: 20.03.2009
Beiträge: 17900

Beitrag TomS Verfasst am: 19. Jul 2013 13:46    Titel: Antworten mit Zitat

Du bringst da was durcheinander.

Betrachten wir ein Volumen V. Die Wahrscheinlichkeit p(V), das Teilchen in diesem Volumen zu finden, erhält man aus dem Integral über genau dieses V



Die Wahrscheinlichkeit, das Teilchen irgendwo (im gesamten Raum) zu finden, ist logischerweise Eins



D.h. die Wahrscheinlichkeitsdichte muss integrierbar (die Wellenfunktion somit quadratintegrierbar) sein.

Daraus kann man jedoch nicht zwingend folgern, dass die Wellenfunktion im Unendlichen verschwindet. Es gibt pathologische, jedoch m.E. unphysikalische Lösungen, wo dies nicht zutrifft.

volvic hat Folgendes geschrieben:
... dass das Teilchen immer in einem finiten Bereich gefunden werden muss (das Integral 1 ergibt).

Es muss kein endlicher Bereich sein.

volvic hat Folgendes geschrieben:
... an manchen Orten wird das Teilchen nicht zu finden sein auf Grund der Interferenz. Das Integral ist somit nicht 1, da das Teilchen nicht überall zu finden ist?

Es geht um das Integral über das Betragsquadrat der Wellenfunktion, nicht einfach nur um die Wellenfunktion.

_________________
Niels Bohr brainwashed a whole generation of theorists into thinking that the job (interpreting quantum theory) was done 50 years ago.
Volvic
Gast





Beitrag Volvic Verfasst am: 20. Jul 2013 01:32    Titel: Antworten mit Zitat

Danke für deine Antwort. Ja ich habe da wohl was vertauscht, da ich nicht geschaut habe über welchen Bereich integriert wird.

Aber etwas verstehe Danke ich immer noch nicht. Zusätzlich habe ich mich in meinem Post falsch ausgedrückt und es ist mir klar, dass es um das Integral des Betragsquadrat geht.



Wie Du gesagt hast, kommt man damit zur Aussage, das das Teilchen irgendwo (im gesamten Raum) zu finden sein muss.

Was ich nun nicht verstehe ist, dass zum Beispiel bei einem Doppelspaltversuch das Teilchen nicht unbedingt auffindbar ist, da es zum Beispiel an einem Ort mit destruktiver Interferenz theoretisch nicht auffindbar wäre.

Somit wäre ja das Betragsquadratintegral nicht 1, da das Teilchen nicht zwingend irgendwo im gesamten Raum gefunden wird.

Für eine aufschlussreiche Antwort wäre ich dankbar.
jh8979
Moderator


Anmeldungsdatum: 10.07.2012
Beiträge: 8571

Beitrag jh8979 Verfasst am: 20. Jul 2013 03:44    Titel: Antworten mit Zitat

Volvic hat Folgendes geschrieben:

Somit wäre ja das Betragsquadratintegral nicht 1, da das Teilchen nicht zwingend irgendwo im gesamten Raum gefunden wird.

Doch, integriert über den gesamten Raum ist es 1. Das Teilchen wird ja irgendwo auf dem Schirm auftreffen.

Wenn Du allerdings nur über ein Gebiet ganz nah am Interferenzmininum integrierst, dann erhaelst Du in der Tat nicht 1, sondern etwas sehr kleines nahe bei 0.
TomS
Moderator


Anmeldungsdatum: 20.03.2009
Beiträge: 17900

Beitrag TomS Verfasst am: 20. Jul 2013 07:52    Titel: Antworten mit Zitat

Die destruktive Interferenz führt nicht dazu, dass man des Teilchen nicht mehr findet (nirgendwo mehr findet), sondern das man es an bestimmten Stellen nicht mehr findet. Dafür findet man es eben an anderen Stellen vergleichsweise häufiger.

Interferenz und ebene Wellen sind deswegen eher ein ungeeignetes Beispiel, da letztere gerade nicht quadratintegrierbar sind.

_________________
Niels Bohr brainwashed a whole generation of theorists into thinking that the job (interpreting quantum theory) was done 50 years ago.
Volvic
Gast





Beitrag Volvic Verfasst am: 20. Jul 2013 13:29    Titel: Antworten mit Zitat

jh8979 genau das meine ich.

Wenn man nahe beim Interferenzminimum integrieren würde, ginge das Integral gegen 0. Und wenn das Integral an manchen Orten gegen 0 geht, kann meiner Ansicht nach das Geamtintegral nicht gegen 1 gehen.

Und somit ist nicht zu jedem Zeitpunkt das Teilchen im Raum zu finden.

Müsste man dann über dtdr integrieren um dies richtig darzustellen?

Ich mein, ich versteh wieso man das Integral gleich 1 setzt, da eben das Teilchen immer irgendwo auf dem Schirm gefunden werden kann. Aber wenn man den ganzen Raum zu jedem Zeitpunkt betrachten würde im Betrweil es zu manchen Zeitpunkten zu bestimmten Zeitpunkten nicht aufzufinden wäre. agsquadrat Integral, dann würde das Integral meines Erachtens nicht 1 ergeben.
TomS
Moderator


Anmeldungsdatum: 20.03.2009
Beiträge: 17900

Beitrag TomS Verfasst am: 20. Jul 2013 14:14    Titel: Antworten mit Zitat

Volvic hat Folgendes geschrieben:
Wenn man nahe beim Interferenzminimum integrieren würde, ginge das Integral gegen 0. Und wenn das Integral an manchen Orten gegen 0 geht, kann meiner Ansicht nach das Geamtintegral nicht gegen 1 gehen.

Die Interferenzminima liegen in einigen Beichen im Raum. Die Maxima in anderen. Man integriert aber über den gesamten Raum. Und man definiert die Normierung der Wellenfunktion genau so, dass das o.g. Integral über den gesamten Raum exakt Eins ist (hätte man also eine Wellenfunktion, für die das nicht gilt, dann multipliziert man diese mit einer geeigneten Konstanten, so dass dies wieder gilt; diese Normierung auf Eins ist eine Grundvoraussetzung und geht letztlich in die Definition der Wellenfunktion ein).

Nun betrachten wir die Zeitentwicklung einer Wellenfunktion. Diese ist durch einen unitären Operator gegegen, also durch eine "Drehung im Hilbertraum". D.h. dass die Normierung gleich Eins unter der Zeitentwicklung exakt erhalten bleibt, also für alle Zeiten gilt.

_________________
Niels Bohr brainwashed a whole generation of theorists into thinking that the job (interpreting quantum theory) was done 50 years ago.
Volvic
Gast





Beitrag Volvic Verfasst am: 20. Jul 2013 22:35    Titel: Antworten mit Zitat

Ok ich verstehe diese Art der Definition.

Trotzdem empfinde ich es immer noch als falscher Ansatz. Oder ich verstehe immer noch etwas nicht.

Du sagst man definiert die Normierung der Wellenfunktion genau so, dass das Integral über dem gesamten Raum genau 1 ergibt.

Wenn ich es richtig verstehe, definiert man das auf diese Weise, damit man sagen kann, dass das Teilchen irgendwo mit Wahrscheinlichkeit 1 (zu 100%) im Raum gefunden werden kann. Und ich verstehe nicht, wieso dieser Schritt überhaupt notwendig ist und es erscheint mir intuitiv falsch.

Deshalb verstehe ich nicht, wieso diese Definition eine Grundvoraussetzung ist für die Normierung der Wellenfunktion.

Um mein Unverständnis noch einmal zusammenzufassen:

1. Man hat einen Versuchsaufbau zB Doppelspalt.

2. Man versucht ein passendes mathematisches Model aufzustellen um das Versuchsergebnis darzustellen.

3. Wellenfunktionen sind passend, da sie an manchen Orten Minima und Maxima mit sich bringen.

4. An Orten von Minima in der Wellenfunktion sollte das Teilchen nicht aufzufinden sein. Was die Funktion schon ohne Normierung mit sich bringt (zum Beispiel wenn die Funktion 0 ergibt für den Ort).

5. Und dann plötzlich normiert und integriert man die Funktion um zur Aussage zu gelangen, dass das Teilchen zu 100% im Raum zu finden ist? Was aber an Orten von Minima nicht der Fall ist.

Meiner Ansicht nach gelangt man mit dieser Art der Definition der Normierung und dem Integral zu einer Faschaussage. Von mir ausgesehen wiederspricht 5. dann 4. und ich sehe nicht aus welchem Grund man dies so tut.
TomS
Moderator


Anmeldungsdatum: 20.03.2009
Beiträge: 17900

Beitrag TomS Verfasst am: 20. Jul 2013 23:07    Titel: Antworten mit Zitat

Volvic hat Folgendes geschrieben:
Du sagst man definiert die Normierung der Wellenfunktion genau so, dass das Integral über dem gesamten Raum genau 1 ergibt.

Wenn ich es richtig verstehe, definiert man das auf diese Weise, damit man sagen kann, dass das Teilchen irgendwo mit Wahrscheinlichkeit 1 (zu 100%) im Raum gefunden werden kann. Und ich verstehe nicht, wieso dieser Schritt überhaupt notwendig ist und es erscheint mir intuitiv falsch.

Nun, man weiß, dass man ein einzelnes Teilchen betrachtet, das nun mal da ist, also irgendwo existiert. Damit muss man entsprechend normieren, und die Normierung muss in der Zeit erhalten sein; das Teilchen kann nicht einfach verschwinden

Volvic hat Folgendes geschrieben:
1. Man hat einen Versuchsaufbau zB Doppelspalt.

2. Man versucht ein passendes mathematisches Model aufzustellen um das Versuchsergebnis darzustellen.

3. Wellenfunktionen sind passend, da sie an manchen Orten Minima und Maxima mit sich bringen.

4. An Orten von Minima in der Wellenfunktion sollte das Teilchen nicht aufzufinden sein. Was die Funktion schon ohne Normierung mit sich bringt (zum Beispiel wenn die Funktion 0 ergibt für den Ort).

5. Und dann plötzlich normiert und integriert man die Funktion um zur Aussage zu gelangen, dass das Teilchen zu 100% im Raum zu finden ist? Was aber an Orten von Minima nicht der Fall ist..

4. ist korrekt. Die Normierung ändert auch nichts daran, dass ein Teilchen nie an der Stelle mit Wsk. 0 gefunden wird, aber es legt den Wert an anderen Stellen fest.

Bei 5. widersprichst du dir selbst: "dass das Teilchen zu 100% im Raum zu finden ist" meint den gesamten Raum, aber "was aber an Orten von Minima nicht der Fall ist" meint einen bestimmten Bereich.

Die Wahrscheinlichkeit, dass bei einem Würfel irgendeine der Zahlen 1..6 oben liegt ist 1, die Wahrscheinlichkeit für eine bestimmte Zahl, z.B. für die 6, ist 1/6, die Wahrscheinlichkeit für die 7 ist 0. Aber trotzdem ist die Gesamtwahrscheinlichkeit auf 1 normiert. Wenn du anders normieren würdest, dann wäre die Wsk. für die 7 immer noch korrekterweise 0, die für eine andere Augenzahl aber c/6, und die für eine beliebige Augenzahl wäre c statt 1. das ist aber unsinnig.

Volvic hat Folgendes geschrieben:
Meiner Ansicht nach gelangt man mit dieser Art der Definition der Normierung und dem Integral zu einer Faschaussage. Von mir ausgesehen wiederspricht 5. dann 4. und ich sehe nicht aus welchem Grund man dies so tut.

Nun, ich kann es evtl. nicht besser erklären als oben: das Teilchen ist mit Sicherheit irgendwo, also muss die Wsk. es zu finden, wenn man überall nachschaut, gleich 1 sein.

Wenn du sicher weißt, dass deine Uhr in deiner Wohnung ist, dann ist die Wahrscheinlichkeitsdichte für deine Uhr integriert über deine Wohnung gleich 1, auch wenn die Wahrscheinlichkeitsdichte in einen bestimmten Bereich, z.B. in einer Bierflasche auf deinem Tisch, 0 ist. Wenn du nun eine komplizierte Wechselwirkung anschaltest, die alles in deiner Wohnung durcheinanderwürfelt, ohne etwas kaputt zu machen, dann ist die Wahrscheinlichkeitsdichte integriert über die Wohnung immer noch 1, die Wahrscheinlichkeitsdichte innerhalb der Bierflasche (die jetzt in deinem Backofen steht) ist immer noch 0.

_________________
Niels Bohr brainwashed a whole generation of theorists into thinking that the job (interpreting quantum theory) was done 50 years ago.
TomS
Moderator


Anmeldungsdatum: 20.03.2009
Beiträge: 17900

Beitrag TomS Verfasst am: 20. Jul 2013 23:30    Titel: Antworten mit Zitat

Volvic hat Folgendes geschrieben:
Was ich nun nicht verstehe ist, dass zum Beispiel bei einem Doppelspaltversuch das Teilchen nicht unbedingt auffindbar ist, da es zum Beispiel an einem Ort mit destruktiver Interferenz theoretisch nicht auffindbar wäre.

Somit wäre ja das Betragsquadratintegral nicht 1, da das Teilchen nicht zwingend irgendwo im gesamten Raum gefunden wird.

Das Teilchen ist sicher irgendwo im Raum, da es nicht einfach verschwinden kann. Es ist nur eben sicher nicht an der Stelle der Minima.

Stell dir die Versuchsanordnung einmal ohne und einmal mit Doppelspalt vor. Durch das Hinzufügen des Doppelspaltes änderst du nichts an der Tatsache, dass das Teilchen da ist, du veränderst nur wo es ist bzw. sein kann.

_________________
Niels Bohr brainwashed a whole generation of theorists into thinking that the job (interpreting quantum theory) was done 50 years ago.
volvic
Gast





Beitrag volvic Verfasst am: 21. Jul 2013 02:40    Titel: Antworten mit Zitat

Vielen Dank für deine auführlichen Antworten. Ich werde diese Morgen noch detaillierter anschauen. Nur noch kurz zu deinem Beispiel:

TomS hat Folgendes geschrieben:
Wenn du sicher weißt, dass deine Uhr in deiner Wohnung ist, dann ist die Wahrscheinlichkeitsdichte für deine Uhr integriert über deine Wohnung gleich 1, auch wenn die Wahrscheinlichkeitsdichte in einen bestimmten Bereich, z.B. in einer Bierflasche auf deinem Tisch, 0 ist.



Wie müsste man den mathematisch vorgehen, wenn ich mir nicht sicher bin ob meine Uhr in meiner Wohnung ist?

Oder wenn ich nur zu 50% sicher bin, dass meine Uhr in meiner Wohnung ist und zu 50%, dass ich sie habe liegen lassen.


Gibt es irgendeinen mathematischen Beweis dafür, dass die Wahrscheinlichkeitsdichte über einem Bereich 1 bleibt, auch wenn sich darin Orte mit Wahrscheinlichkeitsdichte 0 befinden?
TomS
Moderator


Anmeldungsdatum: 20.03.2009
Beiträge: 17900

Beitrag TomS Verfasst am: 21. Jul 2013 08:28    Titel: Antworten mit Zitat

volvic hat Folgendes geschrieben:
Wie müsste man den mathematisch vorgehen, wenn ich mir nicht sicher bin ob meine Uhr in meiner Wohnung ist?

Das ist irrelevant, denn in der QM wird vorausgesetzt, dass das Teilchen da ist. Deswegen wird über den gesamten Raum integriert. Die QM gibt dir sozusagen ein Rezept für die Berechnung der Wsk-dichte bei x, unter der Voraussetzung das das Teilchen mit Wahrscheinlichkeit 1 existiert. Du berechnest die Auftreff-wsk. für ein Teilchen im Experiment auf dem Schirm. Du berechnest die Wsk. für ein Elektron in einem H-Atom in einem bestimmten Zustand. Du setzt aber immer voraus, dass des alles existiert.

Die Wsk., mit einem fairen Würfel ein 6 zu würfeln beträgt 1/6. Die Existenz des Würfels wird vorausgesetzt. Wenn der Würfel nicht da ist, ist das Experiment ziemlich witzlos.

volvic hat Folgendes geschrieben:
Gibt es irgendeinen mathematischen Beweis dafür, dass die Wahrscheinlichkeitsdichte über einem Bereich 1 bleibt, auch wenn sich darin Orte mit Wahrscheinlichkeitsdichte 0 befinden?

Zunächst mal ist kein Beweis nötig, sondern nur eine Konstruktion bzw. Definition. Du startest mit einer Wellenfunktion (üblicherweise einer Lösung der Schrödingergleichung). Du normierst diese so, dass das Integral der Wsk-dichte über den gesamten Raum 1 ergibt. Das funktioniert immer, außer wenn die Wellenfunktion identisch Null das (das wird ausgeschlossen). Fertig.

Mit was hast du nun ein Problem? Dass ein Integral über eine Funktion mit Nullstellen evtl. Null wird? Das Absolutquadrat der Wellenfunktion ist positiv semidefinit, also ist das Integral positiv, also kann es auf Eins normiert werden. Dass das Teilchen im Zuge des Experimentes verschwinden könnte? Der Formalismus der QM garantiert, dass des nicht erfolgt. Der Zeitenwicklungsoperator ist unitär, d.h. Norm-erhaltend, d.h. die Normierung auf 1 ändert sich nicht mit der Zeit. Und für die Wahrscheinlichkeitsstromdichte gilt als Konsequenz des Noethertheorems eine Kontinuitätsgleichung, d.h. dass as eine Bereich abfließende Wsk. in einem benachbarten Bereich hinzukommen muss

(diese letzten beiden Punkte bzgl. Zeitentwicklung und Kontinuitätsgleichung wären einen Beweis hier wert; ich mach mir die Arbeit aber nur dann, wenn es genau die Punkte sind, die dir weithelfen)

Darf ich fragen, wo du bzgl. deiner Kenntnisse in Mathe und Pysik stehst? Ich habe den Eindruck, dass deine Probleme eigtl. nichts mit QM zu tn haben.

_________________
Niels Bohr brainwashed a whole generation of theorists into thinking that the job (interpreting quantum theory) was done 50 years ago.
Volvic
Gast





Beitrag Volvic Verfasst am: 21. Jul 2013 16:12    Titel: Antworten mit Zitat

Ja mein Unverständnis mit der QM liegt wahrscheinlich wirklich anderswo und wir reden somit aneinander vorbei. Vielleicht hilft mir folgendes Beispiel mit dem Doppelspalt.

Nehmen wir an der Laser lässt immer nur ein Teilchen alle 10s raus. Alle diese Teilchen kommen dann auch auf dem Schirm an. Was zu der bekannten Aussage führt, dass das Betragsquadrat der Wellenfunktion 1 ergeben muss. Soweit richtig?

Aber nun mein Unverständnis. Sollte dann nicht irgendwann laut der Wellentheorie ein Teilchen rauskommen, das an einem der weissen Punkten vernichtet wird und nicht auf dem Schirm ankommt?

Oder ist genau dass der Grund, wieso man normiert? --> Um den Sachverhalt darzustellen, dass jedes Teilchen das rausgeht aus dem Laser auch auf dem Schirm ankommt?

Ich glaub mit meinem letzten Satz hab ich mir wohl gerade selbst beigebracht, wieso man normiert. smile.

Ich hatte Vorlesungen in PMM, Mechanik, Relativitätstheorie und QM. In mathe alg1,2 und lin alg 1 und 2.

Die beiden Beweise zu Zeitentwicklung und Kontinuitätsgleichung würden mich natürlich immer noch interessieren.

Ich denke mir ist dank diesem Thread einiges klar geworden bezüglich QM. Sonst würde ich mich wieder melden. Vielen Dank.
D2



Anmeldungsdatum: 10.01.2012
Beiträge: 1723

Beitrag D2 Verfasst am: 21. Jul 2013 16:54    Titel: Antworten mit Zitat

Volvic hat Folgendes geschrieben:
Nehmen wir an der Laser lässt immer nur ein Teilchen alle 10s raus. Alle diese Teilchen kommen dann auch auf dem Schirm an. Was zu der bekannten Aussage führt, dass das Betragsquadrat der Wellenfunktion 1 ergeben muss. Soweit richtig?

Aber nun mein Unverständnis. Sollte dann nicht irgendwann laut der Wellentheorie ein Teilchen rauskommen, das an einem der weissen Punkten vernichtet wird und nicht auf dem Schirm ankommt?


Lese bitte diese Aussage noch ein Mal
TomS hat Folgendes geschrieben:
Die destruktive Interferenz führt nicht dazu, dass man des Teilchen nicht mehr findet (nirgendwo mehr findet), sondern das man es an bestimmten Stellen nicht mehr findet. Dafür findet man es eben an anderen Stellen vergleichsweise häufiger.

Eine Vernichtung findet nicht statt!
Sendest du n Teilchen, findest du auch alle n Teilchen wieder nur nicht an allen Stellen des Schirms.

_________________
Lösungen gibt es immer, man muss nur darauf kommen.
Volvic
Gast





Beitrag Volvic Verfasst am: 21. Jul 2013 22:09    Titel: Antworten mit Zitat

Ja, danke. Ich denke ich hab viel über QM gelernt in diesem Thread.
TomS
Moderator


Anmeldungsdatum: 20.03.2009
Beiträge: 17900

Beitrag TomS Verfasst am: 22. Jul 2013 08:54    Titel: Antworten mit Zitat

TomS hat Folgendes geschrieben:
Mit was hast du nun ein Problem? Dass ein Integral über eine Funktion mit Nullstellen evtl. Null wird? Das Absolutquadrat der Wellenfunktion ist positiv semidefinit, also ist das Integral positiv, also kann es auf Eins normiert werden. Dass das Teilchen im Zuge des Experimentes verschwinden könnte? Der Formalismus der QM garantiert, dass des nicht erfolgt. Der Zeitenwicklungsoperator ist unitär, d.h. Norm-erhaltend, d.h. die Normierung auf 1 ändert sich nicht mit der Zeit. Und für die Wahrscheinlichkeitsstromdichte gilt als Konsequenz des Noethertheorems eine Kontinuitätsgleichung, d.h. dass as eine Bereich abfließende Wsk. in einem benachbarten Bereich hinzukommen muss

Sind die die Argumente klar?

Zur Zeitentwicklung eine ganz kurze Beweisskizze. Gegeben sei ein Zustand psi zur Zeit t sowie ein Hamiltonoperator H. Die Zeitentwicklung das Zustandes zu einer neuen Zeit t' lautet dann



Der Zeitentwicklungsoperator



ist unitär, d.h.



Dies folgt, da H selbstadjungiert ist



Damit gilt für die Norm eines beliebigen Zustandes



Die Norm eines beliebigen Zustandes ist also unter Zeitentwicklung invariant. Physikalisch kann dies mit Erhaltung der Gesamt-Wahrscheinlichkeit identifiziert werden

_________________
Niels Bohr brainwashed a whole generation of theorists into thinking that the job (interpreting quantum theory) was done 50 years ago.
TomS
Moderator


Anmeldungsdatum: 20.03.2009
Beiträge: 17900

Beitrag TomS Verfasst am: 22. Jul 2013 09:14    Titel: Antworten mit Zitat

Noch eine kurze Skizze zur Kontinuitätsgleichung.

Man kann die zeitabhängige Schrödingergleichung aus einer Lagrangedichte ableiten. Diese Lagrangedichte hat eine globale Symmetrie



Wie üblich folgt aus dieser Symmetrie gem. Noethertheorem ein erhaltener Strom





mit der Kontinuitätsgleichung



Daraus folgt die Erhaltung der Ladung Q, d.h. der Norm



Diese Ladung ist aufgrund der Kontinuitätsgleichung sowie mittels des Satzes von Stokes erhalten; der Wahrscheinlichkeitsstrom durch eine geschlossene Fläche S entspricht der zeitl. Änderung der Wahrscheinlichkeitsdichte innerhalb des von S begrenztem Volumens; für eine Fläche im Unendlichen gilt


_________________
Niels Bohr brainwashed a whole generation of theorists into thinking that the job (interpreting quantum theory) was done 50 years ago.
Volvic
Gast





Beitrag Volvic Verfasst am: 23. Jul 2013 23:07    Titel: Antworten mit Zitat

Danke Tom für all deine Posts. Die Beweise sehen super aus.

Ich werde sie mir wohl am Weekend genauer anschauen.

Die Argumente scheinen mir nun logisch zu sein. Finds cool das es dieses Forum gibt.
TomS
Moderator


Anmeldungsdatum: 20.03.2009
Beiträge: 17900

Beitrag TomS Verfasst am: 23. Jul 2013 23:21    Titel: Antworten mit Zitat

Danke

Prost

_________________
Niels Bohr brainwashed a whole generation of theorists into thinking that the job (interpreting quantum theory) was done 50 years ago.
Neue Frage »
Antworten »
    Foren-Übersicht -> Quantenphysik